what is the answer for x⁵÷x⁸

Answers

Answer 1

answer:

the answer is 40 because any expression divides itself is equals to 1

Step-by-step explanation:

Answer 2

Hi there!  

»»————- ★ ————-««

I believe your answer is:  

[tex]\frac{1}{x^3}[/tex]

»»————- ★ ————-««  

Here’s why:

⸻⸻⸻⸻

[tex]\boxed{\text{Calculating the Answer...}}\\\\x^5\div x^8\\---------\\\rightarrow \text{Rewrite as:} \frac{x^5}{x^8}\\\\\rightarrow \text{Recall the exponent rule: } \frac{x^a}{x^b}=x^{a-b}\\\\\rightarrow \frac{x^5}{x^8} = x^{-3}\\\\\rightarrow \text{Recall the exponent rule: }x^{-b}=\frac{1}{x^b}\\\\\rightarrow x^{-3}\\\\\rightarrow \boxed{\frac{1}{x^3}}[/tex]

⸻⸻⸻⸻

»»————- ★ ————-««  

Hope this helps you. I apologize if it’s incorrect.  


Related Questions

can I get a walkthrough on these problems please?

Answers

Answer:

Step-by-step explanation:

52 + 27 = angle 1 (sum of two interior opposite angle is equal to the exterior angle formed)

79 = angle 1

angle 2 + 71 = 92 degree (sum of two interior opposite angle is equal to the exterior angle formed)

angle 2 = 92 - 71

angle 2 = 21 degree

Please help me please

Answers

Answer:

21

Step-by-step explanation:

If lines m and n are equal, line t intersects both of them at the exact same angle. This means that 6x+3=7x-19. Start moving the variables to one side. 3=x-19. Then move the numbers to the other side of the equation. x=21.

(First correct answer for brainliest) Find the length of JM

Answers

Answer:

JM = 18 units

Step-by-step explanation:

∆MHN and ∆JKH are similar (by A.A.A axiom)

so, their sides are proportional

i.e.

(JM+18)/(20+15) = 18/15

or, JM + 18 = 540/15

or, JM = 36-18

hence, JM = 18

Equations of parallel lines

Answers

Answer: c

Step-by-step explanation:

In order to be parallel, the slope must be the same. This means slope is 2

Now one of the point is 5,-1, so do 2x5 + x = -1, so x = -11

meaning b is -11

so the equation should be y=2x-11

What is the weight of one boxe of a 165 ounces

Answers

Answer:

10 pounds

Step-by-step explanation:

165 ounces = 10 pounds

Answer:

10 pounds, 5 ounces

Step-by-step explanation:

The phrasing of the question is unclear. 165 ounces is itself a measure of weight, but it can be further broken down into pounds and ounces.

Recall that 1 pound = 16 ounces. Therefore, 165 ounces = 10 pounds with remainder of 5 ounces. Or in simple decimals it is 165 ounces / 16 ounces = 10.3125 pounds.


Dan, Harry and Regan sell cars.
Dan sells x cars.
Harry sells 5 more cars than Dan.
Regan sells twice as many cars as Dan.
Write an expression, in terms of x, for the mean number of cars Dan, Harry and Regan sell.

Answers

Answer: (4x + 5)/3

Explanation:

D = x
H = x + 5
R = 2x

Mean = (D + H + R)/3
= (x + x + 5 + 2x)/3
= (4x + 5)/2

Use the number line to find the measure of SV

Answers

Answer:

8

Step-by-step explanation:

S = -7 to V= 1 you count from -7 to 1

Simplify 3a²b³x 4a³b

Answers

The answer will be: 12a^5b^4
The answer is 12a5b4

Can somebody help I’ll give brainless plss

Answers

Answer:

$93.22

Step-by-step explanation:

bill = $79

tip = 18%

tip amount = tip % of bill

=18/100 * $79

=$1422/100

=$14.22

total cost = $79 + $14.22

=$93.22

Answer:

93.22

Step-by-step explanation:

Find the amount of the tip

79*18%

79 * .18

14.22

Add this to the amount of the bill

79+14.22

93.22

PLEASE HELP
what is the amount of an investment $10,000 compounded quarterly for 3 years at a rate of 4 percent?

Answers

Answer:

10000 times 3 times 0.04 = 1200

Answer:

11268.25

Step-by-step explanation:

[tex]P(1+\frac{i}{n})^{n*t}=10000(1+\frac{.04}{4})^{4*3}=11268.2503[/tex]

Each month, Kelsey donates 1/5 of her allowance to her school for supplies. 1/2 of that amount goes to the chorus class. How much of her allowance goes to supplies for the chorus class?
Please show your work.
Thank you :)

Answers

every month 1/10 of her allowance goes to chores class because 1/5x1/2 = 1/10 or 1/2 of 1/5 is 1/10

Question 1
Which of the following numbers is greater than 6 and less than 8?
60
O 780
70

Answers

Answer:

The number which is more than 6 and less than 8 is 7.

Step-by-step explanation:

There are the natural numbers.

To find the number which is more than 6 but less than 8.

The numbers which are more than 6 are 7, 8, 9, 10, ....

The numbers which are less than 8 are ...,3, 4, 5, 6, 7

So, the number which is more than 6 and less than 8 is 7.

(x+1)^2 . (x^2+1) = 0

Answers

Answer:

[tex]x^4+2x^3+2x^2+2x+1[/tex]

Step-by-step explanation:

Given that,

[tex](x+1)^2 . (x^2+1) = 0[/tex]

We know that, [tex](a+b)^2=a^2+b^2+2ab[/tex]

So,

[tex](x+1)^2=x^2+1+2x[/tex]

So,

[tex](x+1)^2 . (x^2+1) = (x^2+1+2x)(x^2+1)\\\\=x^2\times x^2+x^2+2x^3+x^2+1+2x\\\\=x^4+2x^3+2x^2+2x+1[/tex]

So, the value of the given expression is equal to[tex]x^4+2x^3+2x^2+2x+1[/tex]

Perform the transformations

Answers

I can’t see the question it looks blurry

HURRY !!!!!! Which describes the correlation shown in the scatterplot?

Answers

No correlation is shown.

Which similarity postulate or theorem can be used to verify that the two
triangles shown below are similar?

A. SAS theorem
B. SSS theorem
C. AA postulate
D. Similarity cannot be determined.

Answers

Answer:

C

Step-by-step explanation:

AA postulate because we are given 2 of the 3 angles of each triangle. They are both the same in each triangle. Also, we can find the 3rd angle in each triangle. Since both triangles have the exact same angles, they are similar.

The given triangles ΔABC and ΔLMN are similar by AA postulate.

What is a triangle?

A triangle is a three sided polygon that consist of three edges and three vertices .

Here two given triangles are ΔABC and ΔLMN. In these two triangles ∠A=∠L=53° and ∠B=∠M= 72°.

Therefore, the two corresponding angles of triangles ΔABC and ΔLMN are equal . So the third angle of ΔABC is also equal to the corresponding angle of ΔLMN.

Hence two triangles are similar by AA postulate.

Thus two triangles are similar by AA postulate. So option (C) is correct.

To learn more about triagles click here:

https://brainly.com/question/24147586

#SPJ7

Cedarburg's zoo has two elephants. The male elephant weighs 3 3/4 tons and the female
elephant weighs 7/12 of a ton. How much more does the male weigh than the female?

Answers

Answer:

3.16666667 tons

Step-by-step explanation:

hope this helps have a good evening

Martin had 60kgs of sugar. He put the sugar weighing 3/4 kg. How many packets did he fill ??

Answers

Answer:

45kg

Step-by-step explanation:

60.3/4=45

Indicate the method you would use to prove the two A's =. If no method applies, enter "none".
O SSS
O SAS
O ASA
O AAS
ONone

Answers

Answer: I would say SAS

Explanation: The two equal eachother, that would mean that the corresponding angles equal eachother as well as the other side length. Let me know if otherwise.
The answer would be the ASA postulate. The angles where the two triangles intersect in the middle are vertical angles. Vertical angles are congruent. Therefore, the triangles have two pairs of corresponding, congruent angles, and a pair of corresponding, congruent sides. And thus, the ASA postulate proves the two triangles are congruent.

change from improper fraction to mix number 51/7​

Answers

Answer:

7 3/7

Step-by-step explanation:

The money m (in £) Billy has to spend each week is his wage w (in £) subtract the tax t (in £) he pays on his income. Enter a formula for the money he can spend and enter how much he has to spend if he earns £150 a week and pays £42 in tax.

Answers

Answer:

m = w - t

£108

Step-by-step explanation:

m = Money Billy has to spend per week :

m = wage, w - tax, t

Hence,

m = w - t

Given;

Wage = £150 ; tax = £42

m = £150 - £42

m = £108

Solve 4(x - 3) - 2x - 1) > 0​

Answers

Answer:

x > 5.5

Step-by-step explanation:

4(x - 3) - (2x - 1) > 0

4x - 12 - 2x + 1 > 0

4x - 2x - 11 > 0

2x - 11 > 0

2x - 11 + 11 > 0 + 11

2x > 11

2x ÷ 2 > 11 ÷ 2

x > 5.5

Answer:

x>13/2

Step-by-step explanation:

4x-12-2x-1>0

2x-13>0

2x>13

X>13/2

the cost for renting a car is $29.00 a day plus $0.29 for every kilometre after the first 100 km. what is the cost of renting a car after 14 days and 320 km?
(linear equations)

Answers

Answer:

$469.80

Step-by-step explanation:

First, let's find out how many fees would be paid exclusively for renting it out for 14 days.

29.00 * 14 = $406

They have to pay 406 dollars just for having the car for 14 days.

Now let's find out how much will need to paid for driving for 320 km.

We can first take away 100 km because the fee only starts afterwards.

320 - 100 = 220

220 * 0.29 = 63.8

63.80 + 406 = 469.80

Thang decided to borrow ​$90 from his local bank to help pay for a car. His loan was for 3 years at a simple interest rate of 40​%. How much interest will Thang​ pay?

Answers

Answer:

Step-by-step explanation:

P = 90

r = 40% = 40/100 = 0.4

t = 3 years

It is not clear to me what this means. Is the 40% for each year for 3 years or is the interest rate = 40% period. I know that in some of the poorer countries, this is not an unusual amount to pay for each year he holds the loan.

I will take it as 40% for each year for 3 years.

i = prt = 90 * 0.4 * 3

i = 108

Shirts-2-Go sells t-shirts for a base price of $12 per shirt plus a fixed fee of $3 shipping and handling for the whole order. Shirts PLUS sells t-shirts for a base price of $8 per shirt plus a fixed fee of $23 shipping and handling for the whole order. Let x represent the number of shirts ordered and let y represent the total cost of the order.

y = 12x + 3

y = 8x + 23

How many shirts would you have to purchase for the total cost to be the same at both companies?

Answers

Answer:

5 shirts

Step-by-step explanation:

Y=12x+3 equation 1

y=8x+23 equation 2

12x+3=8x+23 substituting y value will cause equations to equal

4x=20

x=5 shirts

check answer

y=12x+3

y=12(5)+3

y=60+3

y=63

y=8x+23

y=8(5)+23

y=40+23

y=63

simplify

x - ( - 6 x )​

Answers

Answer:

7x

Step-by-step explanation:

Answer:

answer

x - ( - 6x )

x + 6x

= 7x

helpppp please!!!!!!

Answers

Answer:

x-8

Step-by-step explanation:

since we dont know how much gas is in the tank, lets make that X.

and since we subtract 8 gallons from how much is in the tank, we make that X-8

Answer:

Inequality:

x < 8

The Graph Would be the Hallow Dot on the 8,  then it would go the left way.

How do you find the equation of a line of a triangle when given the area and one set of coordinates?

Answers

The straight line through two points will have an equation in the form y = m x + c . We can find the value of , the gradient of the line, by forming a right-angled triangle using the coordinates of the two points.

Instructions: Find the slope of the line through the points (-15, 14) and (13, 6). Make sure to simplify
fractions and enter your answers with a foward slash (i.e."/"). If the slope is undefined enter "undefined."
Slope =

Answers

Answer:

= -2/7

Step-by-step explanation:

We can use the slope formula

m = (y2-y1)/(x2-x1)

    = ( 6-14)/(13 - -15)

    = (6-14)/(13+15)

    = -8/28

   = -2/7

I need help very quickly, please!!​

Answers

Answer:

32ft

Step-by-step explanation:

Tan = opp/adj

Tan(65) = n / 15

Tan(65) x 15 = n

n = 32.16 = 32ft

Answer:

32 feet

Step-by-step explanation:

Hi there!

We are given a right triangle and the angle of elevation (one of the acute angles in the triangle) as 65°, as well as one of the legs (sides that make up the right triangle) as 15

We need to find the height of the triangle.

Let's make the height of the triangle (the side we need to find) x

Since we need to find the height of the triangle, let's use tangent. As we have the angle of elevation given, we'll use that.

Recall that tangent is [tex]\frac{opposite}{adjacent}[/tex]

in reference to the angle of elevation, the opposite side is x, and the adjacent side is 15

that means that:

tan(65)=[tex]\frac{x}{15}[/tex]

plug tan(65) into your calculator (make sure it's on degree mode!)

tan(65)≈2.14

2.14=[tex]\frac{x}{15}[/tex]

multiply both sides by 15 to clear the fraction

32.1=x

The problem asks us to round to the nearest foot, so the height of the building is 32 feet

Hope this helps! :)

Other Questions
When we expand (2x + 1/2)^6, what is the coefficient on the x^4 term? 3. DNA and Protein similarities are often used for molecular evidence of evolution. What is being compared to show relatedness among species? Explain how this works. can someone please help me I will mark you as brilliant.The current in a resistor is 5 A and the voltage between its terminals is 40 V. Calculate the resistance. An iron wire has a resistance of 24 . If the voltage across its ends is 12 V, calculate the current in the wire. Question 2 of 10A rectangle's width is 5 feet less than its length. Write a quadratic functionthat expresses the rectangle's area in terms of its length.O A. A(1) = lwB. A(1)=12 + 51C. All) = w(w+5)O D. All) = 12 - 51A rectangle's width is 5 feet less than its length. Write a quadratic functionthat expresses the rectangle's area in terms of its length.O A. A(1) = lwB. A(1)=12 + 51C. All) = w(w+5)O D. All) = 12 - 51 Helppp meee guysssss!! Which two items explain why the Articles of Confederation were considered weak? It did not permit Congress to conduct foreign affairs. It gave too much power to the states. It provided for a very strong federal government. It did not allow Congress to issue currency. Reset Next President Andrew Jackson replaced a number of government workers with his How are dates and times stored by Excel? If a court determines that a manager's corporate decision amounted to self-dealing, a. the manager is automatically personally liable to the corporation. b. the transaction being challenged will be automatically voided. c. the manager will automatically be fired. d. the business judgment rule will not apply. Brenya Estate produces a high quality tea branded Super by blending three types of tea coded A, B and C in the ration 1: 5:1. Originally Type A tea costs GHS 1,600 type B costs GHS 800 and type C costs GHS 1,700 per kg to produce. Brenya Tea Estate packs Super tea in packets of 825g each. Blending and packing costs are 40 per kg. Determine the production cost for a packet of Super tea, Find the difference of (4.2x10^3)-(2.7x10^3) Show work! Given that y is directly proportional to x. If y = 30 and x= 5, write an equation that connects y with x. At the local grocery store, you push a 14.5-kg shopping cart. You stop for a moment to add a bag of dog food to your cart. With a force of 12.0 N you now accelerate the cart from rest through a distance of 2.29 m in 3.00 s. What was the mass of the dog food? My cat ran away. If I get another cat , it will run away. This is an example of which type of reasoning? Which is the graph of f(x) = 2 (4)?540.4)404)4(4,4)333 2222(2.1)6,2)15 -4 -3 -2 -14134-5 4 -3 -2 -14234-5 6 -3 -2 -142345X-2-2 -3-34W4-55Tu54(24) Which of the following is an example of indirect quotation?Select one:a. Torres said to me that he was going to get revenge for hisnephew's death. b. "I thought, Man, I'm going to take care of business," he toldme recentlyc. "That's how I live. I was going hunting. This is my own blood,my nephew."Answer is A. Just got it correct Brook states that the distance on the line is 4 units. Caleb states that the whole line does not have a distance because it continues on forever. Vivian states that the line is 6 units long. A number line goes from negative 8 to positive 8. A line with arrows as endpoints is above the line and contains points L, M, N. A closed circle appears at negative 6 and is labeled L. A closed circle appears at negative 2 and is labeled M. A closed circle appears at positive 4 and is labeled N. Which distance did Brook measure? Which distance did Vivian measure? What type of polynomial is: 2x - 4x^3 - 7 + 6x^2A. quadraticB. linearC. quarticD. cubic Madison and the other framers realized that the role of the new Constitution was to define what is meant by "citizen" and "rights " strictly limit the power of government . spell out the natural rights of citizens . none of the above The Rockwell hardness of a metal is determined by impressing a hardened point into the surface of the metal and then measuring the depth of penetration of the point. Suppose the Rockwell hardness of a particular alloy is normally distributed with mean 70 and standard deviation 3. (Rockwell hardness is measured on a continuous scale.)a. If a specimen is acceptable only if its hardness is between 67 and 75, what is the probability that a randomly chosen specimen has an acceptable hardness?b. If the acceptable range of hardness is (70-c, 70+c) , for what value of c would 95% of all specimens have acceptable hardness?c. If the acceptable range is as in part (a) and the hardness of each of ten randomly selected specimens is independently determined, what is the expected number of acceptable specimens among the ten?d. What is the probability that at most eight of ten independently selected specimens have a hardness of less than73.84? [Hint: Y = the number among the ten specimens with hardness less than 73.84 is a binomial variable; what is p?]